跳到主要内容

标有[不平等]的问题

关于证明、操纵和应用不等式的问题。不要仅仅因为你的问题中出现了不平等就使用这个标签。

30,714问题
筛选依据
排序依据
标记为
0 投票
0 答案
6 意见

$\sin\theta(E,F)=\inf\left\{|E-F|:E\在S_{E}中,F\在F\right\}$中,为什么$\theta?

设$E,F\in\mathcal{G}(V)$。从$E$到$F$的左[0,\frac{\pi}{2}\right]$中的最小角度$\theta(E,F)定义为$$\sin\theta(E,F)=\inf\left\{|E-F|:E\在S_{E}中,F\在F\right\}中$$...
Ann的用户头像
  • 1
投票
1 回答
45 意见

显示$a^2+b^2+c^2\le 9R^2$,其中$R$是边为$a$、$b$、$c的三角形的外半径$

给定三角形$\三角形ABC$,$R$是$\三角形ABC的外半径。证明这一点\开始{align}a^2+b^2+c^2\le 9R^2\tag1\结束{align}我的证明:我们知道$a=2R\在a中$所以$$(1)\...
Hiu Nos的用户头像
0 投票
1 回答
45 意见

${x}^{2}-10x+24<0$的解决方案是什么?$x-4<0$和$x-6>0$如何创建$x∈∅$?你是怎么做区间记数法的?

${x}^{2}-10x+24<0$的解决方案是什么?$x-4<0$和$x-6>0$如何创建$x∈∅$?你是怎么做区间记数法的?这个问题让我很困惑。你将如何转换$x-4<0,。。。
Reelmsy 1的用户头像
0 投票
1 回答
70 意见

如果$2p+3q+r=4,则$pq+pr+qr$的最大值$

设$p,q,r\in\Bbb{r}$如果$2p+3q+r=4$,则求$pq+pr+qr$的最大值。我使用了Cauchy-Schwarz不等式,得到了$p^2+q^2+r^2≥8/7$。但在这之后,我无法再继续了。我觉得。。。
外星人的用户头像
  • 518
0 投票
2 答案
67 意见

如果$a,b,c$是三角形的边,使得$a+b+c=2$,那么证明$ab+bc+ca+abc<1$

如果$a、b、c$是三角形的边,则$a+b+c=2$,然后证明$$ab+bc+ca+abc<1$$我的尝试$b+c>a\右箭头a+b+c>2a \右箭头a<1$。同样,$b<1$和$c<1$$ ...
小牛队的用户头像
  • 9,641
2 投票
0 答案
83 意见

这个组合不等式总是成立吗?

背景在线性代数和矩阵理论中,矩阵特征值的研究是至关重要的。给定矩阵$M$定义为:$$M=xx^\top+yy^\top+zz^\top$$其中$x,y,z\in\mathbb{R}^5。。。
Jasmine的用户头像
投票
2 答案
67 意见

概率中的一个不等式

设$X$和$Y$是满足$E[X^2]的两个随机变量+E[Y^2]<\infty美元。证明$$E[\sqrt{X^2+Y^2}]\geq\sqrt{(E[X])^2+(E[Y])^2}.$$基于上述不等式,...
Alex Nguyen的用户头像
0 投票
1 回答
26 意见

l-2范数是否存在不等式?[已关闭]

以下不等式正确吗?$$E(X^2Y^2)\leq E(X|2)E(Y|2)$$如果这个不等式不正确,我可以加一些常数,使不等式保持为常数吗?
Naive Young的用户头像
-3 投票
0 答案
19 意见

求解下列不等式。用间隔符号写答案。[已关闭]

在此处输入图像描述我根本不知道怎么做这项工作,请帮忙
Leah Carnrike的用户头像
0 投票
0 答案
31 意见

切比雪夫的等式何时成立?

切比雪夫不等式表明,对于任何期望值为$E(X)$且方差为$\text{Var}(X)$$的随机变量$X$$X$偏离其平均值至少$a$有界。。。
Alex Nguyen的用户头像
11 投票
9 答案
403 意见

哪个更大,${\sqrt2}^9$还是$9^{\sqart2}$?(无计算器)

我在想一个经典的问题,“哪一个更大,$e^\pi$还是$\pi^e$?”。。。
Dan的用户头像
  • 27.3公里
0 投票
0 答案
20 意见

$\sum\sqrt{\dfrac{a^4+kb^2c^2}{a^2+kbc}}\geq a+b+c,对于[0;12]中的所有k$

我发现这个链接有问题:https://artproblemsolving.com/community/c6h260824p1418804对于$a、b、c>0$。证明:$$\sum\sqrt{\dfrac{a^4+kb^2c^2}{a^2+kbc}}\geqa+b+c,对于[0;12]中的所有k$$...
Lục Trờng Phát的用户头像
投票
0 答案
82 意见

我如何证明所有$n\geq2的$3(sum_{I=1}^{n}{\lfloor\frac{I^2}{n}\rfloor})-n^2\geq2\$$

OEIS序列A175908表示从该公式导出的序列:使用Legendre$\operatorname{L}$-Function,我能够证明对于$4k+1$形式的素数$n$,值。。。
추민서's用户头像
投票
2 答案
82 意见

证明$\sum\limits_{i=1}^{N}m_i\c_i\\ln\left[\frac{\sum\limits_{i=1}^}N}m.i\c_i \T_i}{T_i\sum\limits_{i=1}^{N}m_i\c_ic}\right]>0$?

我对解决一个相当奇怪的不等式感兴趣,就像我之前的一些帖子一样,这个不等式与物理有关,因为它来自一个必须由数学满足的物理条件。。。
Bml的用户头像
投票
1 回答
65 意见

“$x_n>y_n\text{表示任意大}n\Leftrightarrow\limsup\limits_{n\to\infty}\frac{x_n}{y_n}>1$”是真的吗?

设$\{x_n\}_{n=1}^\infty\subseteq(0,\infty)$和$\{y_n\}_{n=1{^\inffy\substeq(0)$是一些正的(可能是无界的)序列。我试着“干净”和“紧紧&…”。。。
无线索数学专家的用户头像

15 30 50 每页